Solid Shape Volume by Disk Method Problem

Click For Summary
The problem involves finding the volume of a solid with an equilateral triangle base and square cross-sections perpendicular to the x-axis. The initial setup includes defining functions for the triangle's sides and calculating the area of the cross-section. The attempted integration from 0 to a yielded an incorrect volume of (4a^3)/9, while the correct volume is (sqrt(3)*a^3)/6. Feedback indicates that the limits of integration were not correctly determined, suggesting a need for a visual representation of the solid to clarify the integration bounds. Accurate limits are crucial for solving volume problems involving geometric shapes.
harrietstowe
Messages
46
Reaction score
0

Homework Statement


The base of a certain solid is an equilateral triangle of side a, with one vertex at the origin and an altitude along the x-axis. Each plane perpendicular to the x-axis intersects the solid in a square cross section with one side in the base of the solid. Find the volume.


Homework Equations





The Attempt at a Solution



I am pretty unsure about this problem but what I did was:
Let a be one side of this equilateral triangle
Let f(x) = (c*x)/(a*sqrt(3))
Let g(x) = (-c*x)/(a*sqrt(3))
so a*sqrt(3) is the altitude and c is a constant
Solve for c:
tan(60 deg)= (a*sqrt(3))/c
c = a
Sub that into f(x) and g(x)
f(x) = (sqrt(3)*x)/3
g(x) = (-sqrt(3)*x)/3
The next thing I did was to subtract the two functions to get the area
A=f(x)-g(x)= (2*sqrt(3)*x)/3
the lower limit of integration will be 0
the upper limit of integration will be a
So I took the integral (with respect to x) from 0 to a of the Area squared and I got (4a^3)/9
The correct answer though is (sqrt(3)*a^3)/6
I hope my set up wasn't completely off but I guess you guys can give me so feedback.
Thanks
 
Physics news on Phys.org
The integration limit is not correct. If you made a drawing and saw the picture of the body you would have found out the correct limits of integration.

ehild
 
Question: A clock's minute hand has length 4 and its hour hand has length 3. What is the distance between the tips at the moment when it is increasing most rapidly?(Putnam Exam Question) Answer: Making assumption that both the hands moves at constant angular velocities, the answer is ## \sqrt{7} .## But don't you think this assumption is somewhat doubtful and wrong?

Similar threads

  • · Replies 5 ·
Replies
5
Views
2K
  • · Replies 4 ·
Replies
4
Views
1K
  • · Replies 9 ·
Replies
9
Views
2K
Replies
4
Views
2K
Replies
2
Views
1K
  • · Replies 7 ·
Replies
7
Views
1K
  • · Replies 8 ·
Replies
8
Views
4K
Replies
2
Views
2K
  • · Replies 5 ·
Replies
5
Views
2K
  • · Replies 3 ·
Replies
3
Views
2K